How do I calculate this double integral using a change of variables?

Click For Summary
To calculate the double integral of cos((x-y)/(x+y)) dA over the triangular region R, defined by the points (0,0), (2,2), and (2+π, 2-π), a change of variables is necessary with U = x-y and V = x+y. The challenge lies in determining the bounds for the integral due to the non-standard slopes of the triangle's sides. By mapping the Cartesian coordinates of the triangle's vertices to the uv space, one can find the corresponding pairs (u,v) for each vertex. Once these pairs are established, sketching the resulting region will help in accurately determining the bounds for u and v. This approach will facilitate the calculation of the double integral.
mld993
Messages
1
Reaction score
0
The problem is as follows.

Calculate the double integral of cos ((x-y)/(x+y)) dA over R, where R is the triangle bounded by the points (0,0), (2,2), and (2 + pi, 2 - pi).

I understand that you have to set U = x-y and V = x+y. However, I am having a hard time finding the bounds on the integral because the slope of the lines isn't 1. I can't simply add x to the other side of each equation because of the 2+pi and 2-pi. Any help would be appreciated. Thanks!
 
Physics news on Phys.org
mld993 said:
The problem is as follows.

Calculate the double integral of cos ((x-y)/(x+y)) dA over R, where R is the triangle bounded by the points (0,0), (2,2), and (2 + pi, 2 - pi).

I understand that you have to set U = x-y and V = x+y. However, I am having a hard time finding the bounds on the integral because the slope of the lines isn't 1. I can't simply add x to the other side of each equation because of the 2+pi and 2-pi. Any help would be appreciated. Thanks!
It is a case of mapping the coordinates of the points in Cartesians to uv space. When x=y=0, what are u and v? When x=y=2, what are u and v? Similarly, for x,y=2±π. Once you have pairs (u,v), you can sketch the resulting region and then determine the bounds for u and v.
 
Question: A clock's minute hand has length 4 and its hour hand has length 3. What is the distance between the tips at the moment when it is increasing most rapidly?(Putnam Exam Question) Answer: Making assumption that both the hands moves at constant angular velocities, the answer is ## \sqrt{7} .## But don't you think this assumption is somewhat doubtful and wrong?

Similar threads

  • · Replies 3 ·
Replies
3
Views
2K
Replies
2
Views
2K
Replies
3
Views
2K
  • · Replies 19 ·
Replies
19
Views
2K
Replies
5
Views
2K
  • · Replies 2 ·
Replies
2
Views
2K
  • · Replies 2 ·
Replies
2
Views
1K
  • · Replies 3 ·
Replies
3
Views
1K
Replies
21
Views
2K
Replies
10
Views
2K